If you're seeing this message, it means we're having trouble loading external resources on our website.

If you're behind a web filter, please make sure that the domains *.kastatic.org and *.kasandbox.org are unblocked.

Main content

Course: LSAT > Unit 1

Lesson 5: Analytical Reasoning – Worked examples

Ordering setup | Given info–cannot be true 2 | Worked example

Example video showing how to approach a "Given info: cannot be true" question on an ordering setup from the analytical reasoning section of the LSAT.

Want to join the conversation?

Video transcript

- [Instructor] Before you tackle this question don't forget to watch the setup video in which we created the initial diagram that you see here. So the question is which one of the following can not be the monument that was begun in 603? So we're looking for a monument that can't ever be in 603 based on the rules of the setup. Looking at our initial diagram we marked that H couldn't be in 603. Unfortunately, that isn't one of the choices. So that means that there's a deduction that we could have made up front, but we didn't. And if you recall in the setup video, I told you that everything is solvable even if you miss a deduction here or there. So we're gonna see that concept in action right now. We're going to test each choice quickly with our pencil, and then only one of the choices will make us break the rules, that will be our answer. Choice A is S, can S be in 603? Well let's try it. If S is in 603 then GM would be the pair that's in 601. We could make L be second, F be fourth and H be fifth, and that works just fine. B is M, can M be in 603? Well if M is in 603 then that forces GS to be the pair that's first. L could be second, F could be fourth and H could be fifth in this situation, so this is fine too. C, can L be in 603? Well if we put L in 603, lots of possibilities are open to us, we just have to make one of them work and we can move onto the next choice. Let's put G and S into one, M into two, F into four and H into five. Okay, that works so we can move on. D, can G be in 603? Well if we put G into 603, that would put M and F as the pair in one. L would have to be fourth in order to be after G and then F would have to be fifth in order to be later than L. Well, there's no room for H now. And we could have seen that in our initial setup but as you can see, it's not a terribly huge deal that we didn't. So we don't have to beat ourselves up, it certainly may have saved some time to see initially that G can't be later than 602 due to being followed by L, F and H in some order. But you know what, we solved this just fine anyway. And as a bonus we now know that G can never be in 603, so for the remainder of the questions we can keep this new deduction in mind. For the record, E is wrong because F can be in 603. It could be GM in one, L in two, F in three, S in four and H in five. So the answer here is D.